I've been watching your videos for two months now, and for the first time, I was able to solve everything on my own without looking at any hints. And my solution was the same as yours! I'm so happy.
@peterpiispanen614528 күн бұрын
Very inspirational... as a mathematics teacher myself I can truly appreciate the enthusiasm, structure, solution and pedagogics... good work! 😅 PS. Learning something new with every video! DS.
@rhc-weinkontore.k.71189 күн бұрын
Lots of fun listening to you. Thank you for sharing your thoughts.
@philipframpton9428Ай бұрын
You can more quickly find the roots by inspection rather than trial and error. A positive sum means r1 is 4 or higher, since r2,r3 cannot be zero and must be at least -1,-1. An odd remainer from r1+r2+r3 with r1 being even means one of r2/r3 is odd, -1 is the only option. r1 + r3 = 2, the only 2^n pair with a difference of 2 are 4,2. So r1=4, r2=-1, r3=-2.
@cyberaguaАй бұрын
Good solution 👍 *r₁ = 4, r₂ = −2, r₃ = −1* (just checking my sophisticated Gboard with subscript digits 😊)
@topquark22Ай бұрын
Great job. I do have a master's degree in mathematics, but you do a much better job of explaining this stuff to young people than I could ever do.
@cyberaguaАй бұрын
In the general case, if not a natural number, *_n_* can also be either a real transcendental number or a complex transcendental number (as follows from the Gelfond-Schneider theorem). Example: *r₁ = 5, r₂ = −3, r₃ = −1,* so that *2ⁿ = 15* and *n = log₂15* is a real transcendental (i.e. not algebraic) number.
@andrasferencz794813 күн бұрын
Beautiful! Another reason for no other combination: One of the roots must be odd, else we won't get the odd sum. The only odd power of two is one. So one root must have the magnitude 1. The positive number's magnitude must be greater than the sum of the negative ones, Therefore the odd root is -1. Once we fixed this, the other two roots' magnitude must be two apart, else the sum can't be 1. Only solution is 4 and -2. I hope it made sense.
@glorrinАй бұрын
It is a great video But I want to have fun and since it was not written in the question, I want to consider n could be not natural. Then there are infinitely many solutions Let's give an exemple : r1 = 8, r2= -4, r3=-3 2^n = 8*(-4)*(-3) =2^(3)*2^(2)*2^(log2(3)) n = 5+Log2(3) a = -32 -24 + 12 = -44 And if we let n be complex r1 = 1, r2 =1, r3 =-1 r1 + r2 + r3 = 1 2^n = 1 * 1 * -1 =2^(0)*2^(0)*2^(i*Pi /ln(2)) {for those who forgot -1 = e^(i*Pi) = e^(i*Pi *ln2/ln2) = 2^(i*Pi/ln2)} n =i*Pi /ln(2) a = 1 - 1 - 1 = -2
@cyberaguaАй бұрын
Great observation 👍
@cyberaguaАй бұрын
Also some sources claim that *e^(𝒊π)* has infinitely many values, as a multivalued complex exponential function, given by *zʷ = exp(w·Ln(z)) = exp(w·(ln(|z|) + 𝒊·arg(z) + 2𝒊πk)), k ∈ ℤ,* while *exp(𝒊π) = −1* has only one value, as a single-valued function, defined by the corresponding Taylor series *exp(z) = ∑ zⁿ/n!*
@ChristodoulosTsilopoulos-vs7xbАй бұрын
Excellent. Thank you for the presentation.
@Jono4174Ай бұрын
n IS AN INTEGER (or rational, I suppose) otherwise, for example: n = ln(3)/ln(2) a= -5 roots = 3,-1,-1
@cyberaguaАй бұрын
Good point 👍 Prime never said that *_a_* and *_n_* should be integers, and that is an important restriction.
@Grecks75Ай бұрын
@@cyberagua If all 3 roots are integers, a must be an integer, too (by Vieta's theorem). For the same reason, we know that 2^n is an integer. n could in theory be a real number, but it is reasonable and safe to assume that n is meant to be a non-negative integer. Otherwise the problem would not make much sense (there would be infinitely many solutions). Still, it would have been better to include that restriction in the problem statement.
@cyberaguaАй бұрын
@@Grecks75 Yes, *_n_* may also be a real or complex transcendental number, but not algebraic (like *√2̅),* which is itself an interesting math problem (compare with the famous Euler's conjecture regarding logarithms, for example). And yes, it is much better to include the full list of restrictions in the wording of the problem.
@chaosredefined3834Ай бұрын
Since r1, r2 and r3 are factors of 2^n, the only way they can be odd is if they are 1 or -1. And we know that r1 + r2 + r3 = 1. Since the sum of the three roots is odd, either one of them or all three of them are odd. As you argue in the vid, two of them are negative and one of them is positive. Case 1: The positive term is odd, both negative terms are even. So, r1 = 1, r2 = -2^a and r3 = -2^b. This is impossible as 1 + r2 + r3 < 1, but we need it to equal 1. Case 2: One of the negative terms is odd, the other negative term and the positive term are even. Without loss of generality, r2 is odd, r3 is even. So, r1 = 2^a, r2 = -1, and r3 = -2^b for some natural numbers a and b. So, 2^a - 1 - 2^b = 1. Or 2^a - 2^b = 2. So, 2^(a - b)(2^b - 1) = 2. Note that 2^b - 1 has to be odd. The only odd factor of 2 is 1, so 2^b - 1 = 1, and therefore, 2^b = 2, so b = 1. Furthermore, 2^(a-b) = 2, so a-b = 1, and since b = 1, that means a = 2. So, the other factors are 4 and -2. Therefore, a valid set of roots is (4, -1, -2), which gives us 2^n = 8, so n = 3, and a = (4)(-1) + (4)(-2) + (-1)(-2) = -10. Case 3: All terms are odd. Thus, r1 = 1, r2 = r3 = -1. So, r1 + r2 + r3 = 1 + (-1) + (-1) = -1. This contradicts our given info. So, no solution here. Thus, the only solution is n=3, a=-10.
@-wx-78-Ай бұрын
One of negative roots should be -1, otherwise all terms in r₁+r₂+r₃ are even and sum can't be odd. WLOG r₁=2ⁱ, r₂=-2ʲ, r₃=-1: r₁+r₂−1 = 1 r₁−2 = -r₂ 2ⁱ−2 = 2ʲ 2(2ⁱ⁻¹−1) = 2ʲ r₂ can not be -1, hence RHS is even; therefore 2ⁱ⁻¹−1 = 1 ⇒ i = 2 ⇒ r₁ = 4, r₂ = -2, r₃ = -1.
@cyberaguaАй бұрын
Hm, you too are able to type subscripts and superscripts? 🤔 How do you input them? From a smartphone or from a PC or Mac?
@Grecks75Ай бұрын
Good observation. 👍
@-wx-78-Ай бұрын
@@cyberagua On Windows 10+ you can press Win+; and select symbols from palette, or use Alt+KeypadPlus then type Unicode codepoint in hex while holding Alt. My method is somewhat different: custom keyboard layout with true RightAlt and AutoHotkey script with RightAlt hotkeys.
@cyberaguaАй бұрын
@@-wx-78- Thank you for sharing! I'm mostly using a Gboard Dictionary with shortcuts on Android and the Character Pad App by Hussein El Feky to look for symbols: *³√−̅8̅ = 1+𝒊√3̅*
@absolutezero9874Ай бұрын
No reply Still ignoring
@holyshit922Ай бұрын
In Vieta formulas on LHS we have elementary symmetric polynomial and on RHS we have (-1)^{n-k}\frac{a_{n-k}}{a_{n}} Possible roots (4,-2,-1)
@itsphoenixingtimeАй бұрын
For me I recognised that the roots themselves must have absolute values of powers of 2, since the product of the roots is a power of 2. Also, by drawing a graph, there were two possibilities: 1. 3 roots are all + 2. 1 root is +. 2 are - The second case must be true as if all 3 roots are + they cannot sum up to 1 [as the sum would exceed 1 quickly setting the smallest root to 1] So we get that there are 3 roots, 1 positive, 2 negative, and the absolute values of said roots must be equal to a power of 2. For some reason, I decided to try -1,-2 and 4 for an example, and sure enough it matched. Not sure if there was a more rigorous way to do this other than by pure luck. Lol.
@yasinforughi-b1zАй бұрын
I just love watching your videos I hope you'd make more videos on Fourier series Hello from Iran🖐
@simplicity53017 күн бұрын
One more thing you forgot to mention about the roots. r1 + r2 + r3 = 1 1 is an odd integer so two roots must be even and one must be odd. The odd integer cannot be greater than absolute 1 otherwise the roots product will not be a multiple of 2 only.
@babitasingh8620Ай бұрын
We can prove that there is only one solution by extending parity over negative integers. r1=2^d r2=-2^b r3=-2^c[as two roots have to be -ve] 2^d+2^b+2^c=1 If all d,b,c>1 Then LHS is even which is not possible as RHS is odd. d,b or c has to be 0 If d=0, then 1-2^b-2^c=1 -2^b=2^c which is not possible So, b or c=0 Let c=0 2^d-2^b-1=1 2^d-2^b=2 Only possible when d=2 and b=1 So,r1=2^d=4 r2=-2^b=-2 r3=-2^c=-1 Value of r2 and r3 can be interchanged by taking b=0 but it gives same value of a and n. THANK YOU
@Grecks75Ай бұрын
Solution: I assume n is meant to be an integer, otherwise the problem wouldn't make much sense, because there would be infinitely many solutions and answers. Let x1, x2, x3 be the 3 integer roots. By Vieta's theorem, we have: 1) x1 * x2 * x3 = 2^n 2) x1 + x2 + x3 = 1 3) x1*x2 + x2*x3 + x3*x1 = a According to (1), none of the roots can be zero. Being integers they must divide 2^n, and therefore they must themselves be powers of 2 with an optional minus sign. This follows from prime factorization. They can't be all positive, otherwise their sum would be at least 3 which contradicts (2). According to (1) their product is positive; therefore one of them must be positive while the two other roots must be negative. W.l.o.g., let x1 be the positive root and |x2| = 5 which is never a power of 2. We are left with: 2^e1 = 2 + 2^e3. Here we learn that e1 >= 2 and e3 > 0. Dividing by 2 we get: 2^(e1 - 1) = 1 + 2^(e3 - 1). The only way this can be satisfied is with e3 = 1 and e1 = 2. Thus, the 3 integer roots must be x1 = 4, x2 = -1, x3 = -2 (in any order). According to (1) we get: n = e1 + e2 + e3 = 3. According to (3) we get: a = -4 + 2 - 8 = -10. In summary: a = -10, n = 3 ist the ONLY answer. The roots are x1 = 4, x2 = -1, x3 = -2 (in any order). We briefly verify that this satisfies the problem.
@alipourzand6499Ай бұрын
Starting point: from the first equation we know that gcd(r1, r2, r3) must divide 1 and from the third we know that r1, r2, r3 must be powers of 2. So either 1 or -1 is a root.
@karamsedighi24 күн бұрын
I LOVE SOUTH AFRICA , THANKS OF NELSON MANDELA, HE IS EVER ALIVE !!
@pijanV218 күн бұрын
?
@georgelaing257821 күн бұрын
Can you think of a direct way to show that 4abc - b - c (a, b and c are natural numbers) can never be a square integer?
@hervesergegbeto3352Ай бұрын
Thank you very much
@keithrobinson294126 күн бұрын
Have you done a proof of Vieta's formula? I have not heard of it before today.
@williamperez-hernandez396816 күн бұрын
So if we are asked to factorize x^3 - x^2 - 10x - 8, we "instantly" know it is (x-4)(x+2)(x+1). Easy!
@surendrakverma555Ай бұрын
Thanks Sir 🙏🙏🙏🙏
@ryhorabramovich1457Ай бұрын
Because r1, r2, r3 are integer and r1*r2*r3=2^n, then each of |r1|, |r2|, |r3| must be powers of 2 (2^t). Now, from conditions r1+r2+r3=1 and "2 negatives, 1 positive", we can easily find the answer: -2, -1, 4
@0lympyАй бұрын
16:34 I guess, it can be proved from r1 + r2 + r3 = 1 => r1 + r2 = 1 - r3; assume r1=2^n, r2=-2^m, r3=-2^k, where n, m, k >= 1 => RHS is 2^k + 1, which is always odd in these conditions; but LHS is 2^n - 2^m can't be odd!
@PauloDacosta-s1sАй бұрын
I learnt what you call of Vieta’s formulas as Girard equations…..I found out that it was Girard that proved the general theorem and Vietas theorem was proved for positive roots only….so in this case, I think that you should call Girard equation and not Vieta’s formula….what do you think?
@mahoremujini20 күн бұрын
On peut éliminer rapidement le cas r=1: puisque la somme des racines est 1, les deux autres racines sont opposées ; ce qui est en contradiction avec le résultat sur les signes à savoir qu’une seule racine est positive.
@Random_DBL_PlayerАй бұрын
Nahh, this is total brain confusion for me, I'll learn it later when im in class 9 or 10 and me in 8 now
@Grecks75Ай бұрын
That's reasonable. Start with simpler exercises, you can always return to these problems later when you're more experienced. It will all fall into place one day if you keep practicing. I would say his problems are for undergraduates (or for very ambitious highschool students). Not suitable for eighth grade. Btw, the reasoning at the end of this video wasn't particularly good; there are better solution developments in the comments.
@srisaishravan5512Ай бұрын
Day 3 of asking newton sir to restart making videos on integrals
@PeymanMoradi-gv1ikАй бұрын
perfect
@DefbarrАй бұрын
(a,n) = (-4,2) works also
@PrimeNewtonsАй бұрын
Product of roots needs to be positive. This fails
@hervesergegbeto3352Ай бұрын
Bonjour mon professeur
@marcclaude2065Ай бұрын
What's done if n is not a natural number? If n= 1/2.. Vietas does not works.
@ryhorabramovich1457Ай бұрын
r1+r2+r3=2^n As r1, r2, r3 are integer, then 2^n also must be integer
@marcclaude2065Ай бұрын
@@ryhorabramovich1457 why r1, r2, r3 must be integer?
@yuichiro12Ай бұрын
@@marcclaude2065 The question states that the cubic has 3 integer roots.
@DanielKniazАй бұрын
Let r's be 1,2,3, sum=7 which equals 2^n but is not integer. Seems it was missed in preconditions
@cyberaguaАй бұрын
Exactly. In the general case, *_n_* can also be either a real transcendental number or a complex transcendental number (as follows from the Gelfond-Schneider theorem)
@CracksMeUp-18225 күн бұрын
Thanks for the inspirations!
@litekira27Ай бұрын
3:58 I think you forgot to put the link in the description :3
@i18nGuyАй бұрын
First thing I did was to set x = 1, to note that 1 - 1 + a - 2^n = 0, so a = 2^n. Yet you determined a = -10. What is wrong here?
@BP-gn2clАй бұрын
X can not be 1
@BP-gn2clАй бұрын
Or you can say x=1 is not a root of the given eqn
@i18nGuyАй бұрын
I am not claiming 1 is a root only that a is 2^n
@BP-gn2clАй бұрын
@@i18nGuy if xis not equal to 1 then how a=2^n?
@BP-gn2clАй бұрын
@@i18nGuy if you put x=1 in the eqn then RHS is not equal to zero. I hope u understand now
@tontonbeber4555Ай бұрын
This problem is simple ... took me about 2--3 minutes to solve it mentally, without writing anything.